What is the slope for M(4,-8 negative) N(6,-1 negative)

Answers

Answer 1

Answer:

m=7/2

Step-by-step explanation:


Related Questions

how many kilograms of pure water is to be added to 100 kilograms of a 30% saline solution to make it a 10% saline solution

Answers

Answer:

  200 kg

Step-by-step explanation:

If w is the amount of water added, then the concentration of the solution is ...

  salt/solute = 100(0.30)/(100 +w) = 0.10

  30 = 10 +0.10w . . . . . simplify

  20 = 0.10w . . . . . . . . subtract 10

  20/0.10 = w = 200 . . . . divide by 0.10

200 kg of water must be added to dilute the solution to 10%.

hi! can someone please help me with these i’m confused on how to answer it

Answers

Answer:

Step-by-step explanation:

Help! I give brainlest. Problem is in Image

Answers

Answer:

65

Step-by-step explanation:

let's place 9 in place of p and 4 in place of q. I will use a star in place of that symbol for the sake of my convenience having to look for it, apologies.

[tex]9 \star 4 = 9^2-4^2 = 81-16 = 65[/tex]

Answer:

65

Step-by-step explanation:

9^2-4^2=65

Kirk has already prepared 16.27 kilograms of dough and will continue preparing 5.88 kilograms of dough every hour. How many hours did Kirk work if he prepared 80.95 kilograms of dough

Answers

Answer:

11 hours

Step-by-step explanation:

First, you need to subtract 16.27 from 80.95 to get how many kilograms of dough he had prepared in that amount of time. 80.95-16.27=64.68. Then you would divide 64.68 by 5.88 to find the total amount of hours he worked. 64.68/5.88=11 hours of work.

The diagram shows a triangle ABC.
B
Not drawn accurately
12.5 cm
A
4.6 cm
Work out the area of the triangle.
Give your answer to 1 decimal place.


please help and show working

Answers

Answer:

A ≈ 5.8 cm²

Step-by-step explanation:

The area (A) of a triangle is calculated as

A = [tex]\frac{1}{2}[/tex] bh ( b is the base and h the perpendicular height )

Here b = 4.6 and h = 2.5 , then

A = [tex]\frac{1}{2}[/tex] × 4.6 × 2.5 = 2.3 × 2.5 ≈ 5.8 cm² ( to 1 dec. place )

Answer:

Step-by-step explanation:

The hanger image below represents a balanced equation.
27
V
Find the value of v that makes the equation true.
U =
I need the answer

Answers

Answer:

2x =3

The hanger image given in the figure represents balanced equation.

Then we have to write an equation to represent the image.

In balanced form left side = right side

x + x = 1 + 1 + 1

2x = 3

The value v is 9.

What is equation?

An equation is a mathematical statement that shows that two mathematical expressions are equal.

Given that is a structure showing a balanced hanger we need to find the value,

So we can say since the hanger is balanced therefore the weight both sides will be equal,

Therefore,

v + v + v = 27

3v = 27

v = 27

Hence the value v is 9.

Learn more about equation click;

https://brainly.com/question/29657983

#SPJ2

PLS HELP ASAP ILL GIVE BRAINLKEST THANKS

Answers

Answer:

d>-7

Step-by-step explanation:

first answer
d = greater than or equal to 7

A litter of puppies consists of black puppies and white puppies. A

puppy is randomly selected and removed from the litter. Then another

random selection is made from the remaining puppies.

Event A: The first

selection is a black

puppy.

Event B: The second

selection is a white

puppy.

Is A dependent on B?

Answers

We want to see if the first selection is dependent on the second selection, we will see that no, the first selection can not depend on a selection that did not happen yet.

So the situation is:

We have a group of puppies, some are black, some are white.

Let's say that there are N puppies in total, n₁ are black, n₂ are white, such that:

n₁ + n₂ = N

Now we have that the first selection, event A, is a black puppy. The probability is just the quotient between the number of black puppies and the total number of puppies:

q = n₁/N

Then now we have:

N - 1 puppies in total.n₁ - 1 black puppiesn₂ white puppies.

Now for the second selection, event B, the probability of selecting a white puppy is:

P = n₂/(N - 1)

Notice that because there are fewer black puppies than before (because we took one) the denominator decreases, thus, the probability of getting a white puppy increases.

So clearly event B depends on event A.

Now the question is:

Is A dependent on B?

No, A can not depend on event B, because event A happens first. The probability of the first event is always q = n₁/N.

If you want to learn more, you can read:

https://brainly.com/question/12138717

Dave owns 15 shares of ABC Mining stock. On Monday, the value of
each share rose $2, but on Tuesday the value fell $5. What is the
change in the value of Dave's shares?
To travol tho
Write an expression to represent.

Answers

Answer:

15(2) + 15 (-5) . #45 drop

Step-by-step explanation:

Lee earns $17.00 per hour at her job. How much money will Lee earn if she works for 4 hours?

Answers

Lee would earn 68 dollars if she worked 4 hours
He would earn $68 if he worked fo 4 hours

Billy and Michael have 2 lemonade stands. Billy charges $2 per lemonade plus a flat rate of $2.
Michael charges $3 per lemonade plus a flat rate of $1.

How many lemonades would they have to sell to be making the same amount of money?

Answers

1 lemonade.

2x+2=3x+1

-x+2=1
-x=-1
x=1 lemonade

what is the mean number of cars owned by a family? Give your answer to 1 decimal place.

Answers

Answer:

Step-by-step explanation:

total # of families=15

families that own 0 cars=2

families that own 1 cars=5

families that own 2cars=4

families that own 3 cars=3

families that own 4 cars=1

so 2+5+4+3+1=15

total# of families/ sum of cars = 15/15=1

Explain the steps to locate the average rate of change when two points are displayed on an exponential graph. -

Answers

Answer:

Explanation: The rate of change between two points on a curve can be approximated by calculating the change between two points. Let be the coordinates of the first point and be the coordinates of the second point. Then the formula giving approximate rate of change is

Step-by-step explanation:

calculating the change between two points.

ANSWER CORRECT FOR BRAINLIEST

Answers

Answer:

3 2 /5 ×3

change 3 2 /5 into an improper fraction

then you will get 17 /5

after that you say 17/5 ×3/1

=(17×3)/ (5x1)

=51/5 (51÷5)

=10.2/ 10 1/5

Solution:

[tex]3 \frac{2}{5} \times 3 \\ = \frac{17}{5} \times 3 \\ = \frac{51}{5} \\ = 10 \frac{1}{5} [/tex]

Answer:

[tex]10 \frac{1}{5} [/tex]

Hope it helps.

Do comment if you have any query.

5. Do you find similarities between the rules of adding & subtracting dissimilar fractions and
adding & subtracting dissimilar rational algebraic expressions?​

Answers

Answer:

First, we will change dissimilar fractions into similar fractions by finding the least common denominator. Second, add them like similar fractions. And then finally, convert the answer in the lowest term. Example No.6 Subtract 3/4 from 4/5. What will be our least common denominator? The least common multiple of 5 and 4 is 20. That was an Example

Step-by-step explanation:

Mr. Smith charged Jill $320 for parts and $150 per hour for labor to repair her bicycle. If he spent 3 hours repairing her bike, how much does Jill owe him? Write an expression and show your work and answer.

Answers

Answer:A repair bill for a car is $648.45. The parts cost $265.95. The labor cost is $85 per hour.

Step-by-step explanation:

had this question

Could somebody help n explain pls thx

Answers

Answer:

lines are perpendicular

Step-by-step explanation:

• Parallel lines have equal slopes

• The product of the slopes of perpendicular lines = - 1

The equation of a line in slope- intercept form is

y = mx + c ( m is the slope and c the y- intercept )

Rearrange the given equations into this form and extract the slope

3x - 2y = - 6 ( subtract 3x from both sides )

- 2y = - 3x - 6 ( divide through by - 2 )

y = [tex]\frac{3}{2}[/tex] x + 3 ← in slope- intercept form

with slope m = [tex]\frac{3}{2}[/tex]

----------------------------------

4x + 6y = 2 ( subtract 4x from both sides )

6y = - 4x + 2 ( divide through by 6 )

y = - [tex]\frac{4}{6}[/tex] x + [tex]\frac{2}{6}[/tex] , that is

y = - [tex]\frac{2}{3}[/tex] x + [tex]\frac{1}{3}[/tex] ← in slope- intercept form

with slope m = - [tex]\frac{2}{3}[/tex]

--------------------------------------

[tex]\frac{3}{2}[/tex] ≠ - [tex]\frac{2}{3}[/tex] , then lines are not parallel

[tex]\frac{3}{2}[/tex] × - [tex]\frac{2}{3}[/tex] = - 1

Thus the lines are perpendicular to each other

A used car is advertised for sale at "$300 down and $65 a month for 12 months. What is the total price of the car?​

Answers

Answer:

It will be 780

Step-by-step explanation:

explanation is that I multiply 65$ by 12

Use the table to calculate the slope and y-intercept, then write the equation in slope intercept form.

Answers

Answer: y = (28/4)x + 4

Step-by-step explanation:

We'll be using 20, 144 (1) and 24, 172 (2) for this

slope = y2 - y1 / x2 - x1

slope = 172 - 144 / 24 - 20

slope = 28/4

y = mx + b

y = (28/4)x + b

We can then reduce 20, 144 by 4, 28

20, 144

16, 116

12, 88

8, 60

4, 32

0, 4

The y-intercept is when x = 0, so its 4.

Plug that into y = mx + b

y = (28/4)x + 4

the length of 2/3 a rope is (4u-5) inches Express the total length of the rope in terms of u

Answers

Answer:

6u - 15/2 inches

Step-by-step explanation:

(2/3)rope = 4u - 5

multiply both sides by 3/2

rope = 6u - 15/2

a. Why does the provided equation support the fact that there is a horizontal asymptote of F(x) that is not
on the x-axis?
(ax-3)(x-2)divide2(x+3)(x-2)
Y=-2 is a horizontal asymptote

Answers

Applying limits, it is found that since [tex]a \neq 0, \frac{a}{2} \neq 0[/tex], and hence the horizontal asymptote is not y = 0, thus not being on the x-axis.

The function is given by:

[tex]F(x) = \frac{(ax - 3)(x - 2)}{2(x + 3)(x - 2)}[/tex]

The horizontal asymptote is the limit of the function as x goes to infinity, hence:

[tex]y = \lim_{x \rigtharrow \infty} F(x) = \lim_{x \rigtharrow \infty} \frac{(ax - 3)(x - 2)}{2(x + 3)(x - 2)}[/tex]

Considering only the terms with the highest exponents:

[tex]y = \lim_{x \rigtharrow \infty} \frac{ax^2}{2x^2} = \frac{a}{2}[/tex]

Since [tex]a \neq 0, \frac{a}{2} \neq 0[/tex], and hence the horizontal asymptote is not y = 0, thus not being on the x-axis.

For more on horizontal asymptotes and limits, you can check https://brainly.com/question/11598999

Can someone help me with question 8 please this is due tomorrow

Answers

Answer:

New eq: 11x + 1. The line will become steeper.

Step-by-step explanation:

Slope is rise over run. In the original equation, y increased 4 times everytime x increased by one. In the new equation, y increased 11 times everytime x increased by one making it steeper.

I need help pls help me

Answers

Answer:

$1,270

Step-by-step explanation:

let x be the total amount of ice cream sold without tax

x + (x*0.0635)=21270

1.0635x=21270

x=20,000

so the amount he needs to send to state is

21270-20000

1,270

Total cost of icecream without cost be x

[tex]\\ \sf\longmapsto x+0.0635x=21270[/tex]

[tex]\\ \sf\longmapsto 1.0635x=21270[/tex]

[tex]\\ \sf\longmapsto x=21270/1.0635=20000[/tex]

Amount to be paid

[tex]\\ \sf\longmapsto 21270-20000=1270[/tex]

find the limiting value using L hospital​

Answers

Answer:

  -1

Step-by-step explanation:

The expression evaluates to the indeterminate form -∞/∞, so L'Hopital's rule is appropriately applied. We assume this is the common log.

  d(log(x))/dx = 1/(x·ln(10))

  d(log(cot(x)))/dx = 1/(cot(x)·ln(10)·(-csc²(x)) = -1/(sin(x)·cos(x)·ln(10))

Then the ratio of these derivatives is ...

  lim = -sin(x)cos(x)·ln(10)/(x·ln(10)) = -sin(x)cos(x)/x

__

At x=0, this has the indeterminate form 0/0, so L'Hopital's rule can be applied again.

  d(-sin(x)cos(x))/dx = -cos(2x)

  dx/dx = 1

so the limit is ...

  lim = -cos(2x)/1

  lim = -1 when evaluated at x=0.

_____

I find it useful to use a graphing calculator to give an estimate of the limit of an indeterminate form.

What is the slope of this graph please no links or files

Answers

the answer is 2/3 , you go up 2 and to the right 3 which means it is a positive slope of 2/3.

Answer:

2/3

Step-by-step explanation:

You go over 2 and down 3

Lucy has some number cards that each have a whole number on them.
If she chooses a card at random, the probability that the number on it is
even is
3/7

Work out the ratio of even cards to odd cards that Lucy has.
Give your answer in its simplest form.



PLEASE HELP BRAINLIST TO FASTEST AND CORRECT ANSWER

Answers

Answer:

Well lucy has a 50/50 chance to get a odd card so that is it

Step-by-step explanation:

The ratio of even cards to odd cards that Lucy has is 3:4 in its simplest form.

Let's denote the number of even cards as "x" and the number of odd cards as "y".

The probability of choosing an even card is given as [tex]\frac{3}{7}.[/tex]

The probability of choosing an even card is calculated by dividing the number of even cards by the total number of cards:

Probability of choosing an even card = Number of even cards / Total number of cards

Therefore, we have:

[tex]\frac{3}{7} = \frac{x}{x+y}[/tex]

To find the ratio of even cards to odd cards, we can set up the equation:

[tex]\frac{x}{y} = \frac{3}{4}[/tex]

Therefore, the ratio of even cards to odd cards that Lucy has is 3:4 in its simplest form.

To learn more on probability click:

https://brainly.com/question/11234923

#SPJ4

expand and simplify
(x +3)(x+5)

Answers

x • x + x • 5 + 3 • x + 3 • 5
2x + 5x + 3x + 15
10x + 15

Answer:

x² + 8x + 15

Step-by-step explanation:

(x +3)(x+5)

=> x² + 5x + 3x + 15

=> x² + 8x + 15

So, after simplifying, we can conclude that x² + 8x + 15 is our answer. Hoped this helped.

Ruben bought 666 comic books for \$21$21dollar sign, 21. Each comic book was the same price.
What was the cost for 111 comic book?
\$

Answers

Answer:

$3.50

Step-by-step explanation:

111 is a sixth of 666 so its 21/6 which is 3 dollars 50 cents.

Find the perimeter. Simplify your answer.

Answers

Answer:

Step-by-step explanation:

p = 10x + 2 + 7x - 6 + 8x

p = 25x - 4

Answer:

To Find the perimeter we add all the sides,

perimeter = 10x + 2 + 7x - 6 + 8x

= 25x - 4...ans

Step-by-step explanation:

Hope it Helps you!!

what year was it, exactly 4 centuries before 1965?

Answers

Based on the number of years in a century, we can calculate that the 4 centuries before 1965 was 1565.

A century refers to a period of 100 years.

If we are looking for a period of 4 centuries therefore, this would be:

= 4 x 100

= 400 years

4 centuries before 1965 would therefore be:

= 1965 - 400

= 1565

In conclusion, the answer is 1565.

Find out more at https://brainly.com/question/17498856.

Other Questions
PLEASE HELP!! Identify and place the stages of mitosis in the chart. PART B: Which statement offers the strongest support for the answer in Part A?A. "a complete emancipation from all forms of bondage, of custom, dependence,superstition" (Paragraph 1) B. "they must make the voyage of life alone" (Paragraph 1)C. "the possibilities of one human soul will never be found in another (Paragraph 3)D. "each man bears his own burden" (Paragraph 3) discrete packets of kinetic energy in light are called a sum of money at simple interest doubles itself in 10 years .Find the rate percent per annum Match each Animation category to its description how long did it take you to answer all of these questions Does anyone live in noble Oklahoma or was born in noble Oklahoma. What is the acceleration of a 10 kg pushed by a 5N force An art club sells 42 large candles and 56 small candles.a. Use the Distributive Property to write and simplify an expression for the profit. expression for the total profit: _(_-x) + (_-y) simplified expression for the total profit: b. A large candle costs $5, and a small candle costs $3. What is the clubs profit? club's profit: $ Which shows the prodcuts orderd from least to greatest If you have to decide to claim a credit or deduction on your taxes which should you take? Write a program that lists all ways people can line up for a photo (all permutations of a list of strings). The program will read a list of one word names (until -1), and use a recursive method to create and output all possible orderings of those names separated by a comma, one ordering per line. Find the equations of a line through these 2 points(1,2) & (3,5)2. (-2,0) & (0,8)3. (4,-5) & (11,-5)4. (6,2) & (9,3)6. (6,2) & (10,3)5. (7,5) & (6,11) .The implications of medicine to the healthcare system and industry, What is the difference between nightmares and recurringnightmares? If x=5 and y=3 find the value of x+2y 1. Find the slope of the line that passes throughthe points (6, 13) and (-3, 7). When waves are closer together and there are more waves, then the wave has a higher ____________. Kelia, the owner of a Lebanese factory that produces electrical converters, recently learned that the EU will begin taxing all electrical components imported into EU member nations. In this example, a tariff is being implemented to protect European electrical component manufacturers. Group startsTrue or FalseTrue, unselectedFalse, unselected How to write a recommendation letter.